Anticoagulation

Ace your homework & exams now with Quizwiz!

While attempting to manage GH's heart failure with diuretics, his renal function worsened. His BUN and SCr have increased to 36 and 2.1 respectively. GH's physician feels it would be safest to start an unfractionated heparin drip until the renal function stabilizes. He orders a heparin bolus, followed by 18 units/kg/hour continuous infusion. The hospital uses premixed heparin drips that are 25,000 units/250 mL. How many milliliters of heparin will the patient require per hour? 152 lb (Answer must be numeric; no units or commas; include a leading zero when answer is less than 1; round the final answer to the nearest ONE DECIMAL PLACE.)

12.4 ml/hr Heparin is a very old drug and is used off-label much of the time. In most hospitals, the standard VTE/PE dosing is used for all indications requiring systemic anticoagulation, except STEMI. Actual body weight is used for dosing UFH. 18 units/kg/hr x 69.09 kg = 1243.6364 units/hr x 250 mL/25,000 units = 12.436 mL/hr (round to 12.4 as specified in the question).

A new patient is using enoxaparin therapy for "bridging" until her INR is therapeutic. She brings the following over-the-counter medicines to the pharmacy window for payment: DHEA, Women's 50+ multivitamin, Advil Migraine, coenzyme Q10 and a B-Complex vitamin. The pharmacist should offer the following advice: A) Advil Migraine is not safe to use with warfarin; acetaminophen is safer. B) DHEA contains vitamin K and may make the warfarin ineffective. C) Vitamin B complexes cannot be used with warfarin. D) Women's 50+ multivitamin may increase the INR. E) Willow bark may decrease the effectiveness of warfarin.

A) Advil Migraine is not safe to use with warfarin; acetaminophen is safer. NSAIDs, like Advil Migraine, do not raise the INR, but they do increase bleeding risk by an antiplatelet effect. Willow bark contains plant salicylates and can increase the bleeding risk as well.

Which of the following agents is/are safe to use in a patient at risk for HIT? (Select ALL that apply.) A) Argatroban B) Bivalirudin C) Dalteparin D) Enoxaparin E) Heparin

A) Argatroban B) Bivalirudin Argatroban and bivalirudin (Angiomax) are FDA approved for use in patients at risk for HIT. Angiomax is mainly used for PCI.

AG is beginning warfarin therapy. She asks the pharmacist which foods are high in vitamin K. Which of the following foods are high in vitamin K? (Select ALL that apply.) A) Cauliflower B) Canola and soybean oils C) Broccoli and spinach D) Fish and fish oils E) Green and black tea

A) Cauliflower B) Canola and soybean oils C) Broccoli and spinach E) Green and black tea It is important to counsel the patient to eat consistent amounts of vitamin K daily and avoid large, sudden changes in intake of foods rich in vitamin K. These food should not be eliminated from the diet, but consistency is very important.

AZ currently has lung cancer and was diagnosed with a DVT. Her estimated CrCl is 75 mL/min. What anticoagulation therapy would be most appropriate for this patient? A) Edoxaban PO B) Enoxaparin 40 mg SC daily C) Enoxaparin 1 mg/kg SC Q12H D) Warfarin PO E) Xarelto PO

C) Enoxaparin 1 mg/kg SC Q12H According to CHEST and ASCO guidelines, LMWH is recommended over any oral anticoagulant (including warfarin) for treatment of VTE in patients with cancer.

HB has been taking warfarin 6 mg daily for the past 4 months and all of her INR values have been within therapeutic range (INR 2-3) in the past. Today, her INR is 3.2. She has been taking her medication the same way with no additional medications she can recall being recently added to her regimen. What is the most appropriate management of HB's warfarin therapy at this time? A) Continue taking warfarin 6 mg daily and test the INR within 1-2 weeks. B) Hold her warfarin dose until INR is therapeutic, then restart her warfarin back at a dose of 5 mg M, T, Th, F and 6 mg on W, Sat, Sun. C) Hold her warfarin dose until INR is therapeutic, then restart her warfarin back at a dose of 5 mg W, F and Sunday and 6 mg on T, Th, and Sat. and Monday. D) Hold her warfarin dose until INR is therapeutic, then restart her warfarin back at a dose of 4 mg daily. E) Give vitamin K 2.5 mg orally now. Restart warfarin at a dose of 5 mg daily.

A) Continue taking warfarin 6 mg daily and test the INR within 1-2 weeks. The correct recommendation is to continue the current dose and recheck the INR within 1-2 weeks when the patient's previous INRs have been therapeutic and a single reading is within 0.5 of the therapeutic range.

A hospitalized patient is post-op day #1 after a right hip arthroplasty. The patient has a history of a previous VTE. The doctor has recommended that the patient begin warfarin with Lovenox bridge therapy, but the patient responds that he does not wish to take "rat poison". The pharmacy intern wants to explain the risks associated with not taking an anticoagulant in this situation. The intern should explain to the patient that he is at higher risk for the following complications if he chooses not to use anticoagulation: (Select ALL that apply.) A) Deep vein thrombosis B) Pulmonary embolism C) Bleeding D) Pleural effusion E) Cardiac tamponade

A) Deep vein thrombosis B) Pulmonary embolism The primary risks in this patient from not using anticoagulants or having anticoagulants at subtherapeutic levels are deep vein thrombosis (DVT) and/or pulmonary embolism (PE). There are options aside from warfarin that may be acceptable to the patient.

Which of the following procedures can help reduce medication errors associated with heparin? (Select ALL that apply.) A) Do not use the color of the syringe or packaging to verify the dose. B) Provide inservices that review heparin safety. C) If possible, outsource the preparation of heparin flushes. D) Make sure unit nurses prepare the heparin doses. E) Implement protocols to initiate and manage therapy.

A) Do not use the color of the syringe or packaging to verify the dose. B) Provide inservices that review heparin safety. C) If possible, outsource the preparation of heparin flushes. E) Implement protocols to initiate and manage therapy. All anticoagulants are high-alert medications and errors have resulted in patient death. Heparin comes in many different strengths. To help avoid errors, heparin should not be stocked in unfamiliar concentrations. The color of the bag or syringe should not be used to verify the dose. Having the pharmacy prepare the flush syringes (rather than busy unit nurses) can help reduce errors. Buying prepared heparin syringes (outsourcing) is preferable to compounding them on-site. Refer to the Medication Safety chapter for additional discussion of high-alert medications.

What is the generic name of Savaysa? A) Edoxaban B) Apixaban C) Idarucizumab D) Dabigatran E) Betrixaban

A) Edoxaban The generic name of Savaysa is edoxaban.

The ACCP guidelines recommend a lower warfarin starting dose (≤ 5 mg) for the following patient populations: (Select ALL that apply.) A) Elderly or debilitated B) Liver disease C) Heart failure D) Patients with a high risk of bleeding E) Those taking medications such as rifampin

A) Elderly or debilitated B) Liver disease C) Heart failure D) Patients with a high risk of bleeding Rifampin is an inducer and requires a much higher dose, not lower.

KP is a 76 year-old male with a mechanical mitral heart valve, hypertension, type 2 diabetes, and hyperlipidemia. He was recently hospitalized for an elective procedure and discharged with a therapeutic INR of 2.8. Two weeks later he comes back for follow up and his INR is 4.0. Which of the following natural products could have caused an increase in KP's INR? (Select ALL that apply.) A) Glucosamine B) Vitamin E C) Alfalfa D) Ginger E) St. John's wort

A) Glucosamine B) Vitamin E D) Ginger The "5 G's" (garlic, ginger, ginkgo, ginseng and glucosamine) can increase the INR along with many other natural products. St. John's wort (and others) can decrease the INR.

DV has been admitted for a PE. He started heparin and warfarin therapy 3 days ago. Today, it is suspected that he has heparin-induced thrombocytopenia (platelet count is 98,000/mm3). Which of the following statements are true regarding HIT and treatment of this patient? (Select ALL that apply.) A) Heparin should be discontinued immediately. B) Warfarin should be discontinued. C) HIT is defined as a platelet count < 100,000/mm^3 D) Vitamin K should be given E) Platelets should be administered

A) Heparin should be discontinued immediately. B) Warfarin should be discontinued. D) Vitamin K should be given HIT is defined as a > 50% drop in platelet count from baseline. Though HIT is associated with thrombosis (and amputations in some cases), it is important to stop all heparin/LMWH/warfarin and administer vitamin K. Warfarin should not be restarted until platelets have recovered (defined as > 150,000/mm3).

MG is a 43 year-old male who is usually well-controlled on a warfarin regimen of 7.5 mg five days per week, and 5 mg two days weekly. He presents to the anticoagulation clinic to have his INR checked. He reports that he had an upper respiratory infection and the physician had given him a 10-day course of levofloxacin. He just took his last levofloxacin tablet this morning. His INR is elevated today at 3.5; his target therapeutic INR is 2-3. Choose the preferred course of action: A) Hold the warfarin dose today; resume usual dosing regimen when INR is therapeutic and have the patient monitor for symptoms of bleeding. B) Hold warfarin and administer phytonadione 5 mg PO x 1 now. Resume warfarin when the INR is therapeutic. C) Hold warfarin and administer phytonadione 2 mg by SC injection. Resume warfarin when the INR is therapeutic. D) Hold warfarin and administer phytonadione 2 mg by IM injection. Resume warfarin when the INR is therapeutic. E) Hold warfarin and administer phytonadione 2 mg by IV injection. Resume warfarin when the INR is therapeutic.

A) Hold the warfarin dose today; resume usual dosing regimen when INR is therapeutic and have the patient monitor for symptoms of bleeding. Phytonadione is not recommended for INR < 4.5.

JK is a 62 year-old female with chronic urinary tract infections. Several times a year, she receives a prescription for Bactrim. The physician suggested she use the antibiotic daily, but she prefers not to because she feels that she is already using too many medications. JK comes to the pharmacy today with a prescription for warfarin. She tells the pharmacist that the heart doctor found her heart was "beating funny." The pharmacist should emphasize the following counseling to this patient: (Select ALL that apply.) A) If she gets a UTI, the antibiotic Bactrim could make her warfarin level increase. B) When taking both warfarin and Bactrim, separate the doses by 4 hours to decrease the risk of a drug interaction. C) The drug interaction between warfarin and Bactrim for a UTI may lead to significant bleeding. D) Warfarin can cause the Bactrim to be ineffective in treating the UTI. E) She should inform her health care providers that she is using warfarin so they can choose medications that do not interact with warfarin.

A) If she gets a UTI, the antibiotic Bactrim could make her warfarin level increase. C) The drug interaction between warfarin and Bactrim for a UTI may lead to significant bleeding. E) She should inform her health care providers that she is using warfarin so they can choose medications that do not interact with warfarin. Bactrim can inhibit the metabolism of warfarin putting the patient at risk for bleeding. All providers treating the patient should know that she is taking warfarin to prevent drug-drug interactions.

HL is taking Xarelto 15 mg PO BID for a newly diagnosed DVT. Which of the following counseling points should be made to this patient? A) If the patient misses a dose but remembers the same day, take both 15 mg tablets at the same time to ensure that 30 mg is received per day. B) In addition to bleeding, common side effects include paresthesias and muscle weakness. C) The patient should take the Xarelto on an empty stomach. D) The patient can use ginkgo biloba and other herbal medications safely with this medication. E) The patient's blood will need to be tested to measure the effectiveness of this medication.

A) If the patient misses a dose but remembers the same day, take both 15 mg tablets at the same time to ensure that 30 mg is received per day. With BID dosing of Xarelto, both 15 mg tablets should be taken if the patient remembers a missed dose on the same day. This ensures that 30 mg is received per day. Doses ≥ 15 mg of Xarelto must be taken with food.

Select the correct mechanism of action for Pradaxa: A) Oral direct Factor IIa inhibitor B) Injectable direct thrombin inhibitor C) Vitamin K antagonist D) Oral Factor Xa inhibitor E) Inhibits Factor Xa and Factor IIa via antithrombin

A) Oral direct Factor IIa inhibitor Dabigatran (Pradaxa) is an oral direct thrombin (Factor IIa) inhibitor. Direct thrombin inhibitors (which block thrombin, as the name suggests) decrease the amount of fibrin available for clot formation.

A pharmacy receives a prescription for warfarin 5 mg PO daily. What is the color of the warfarin tablet? A) Peach B) Green C) Tan D) Pink E) Blue

A) Peach Warfarin 5 mg tablets are peach.

On day #6 of the hospitalization, ST is doing better. Her chronic conditions are under better control. She mentions to the case manager that she lives almost 2 hours from town and has no transportation to come to the clinic for labs or office visits. The physician would like to discharge ST on an oral anticoagulant that does not require laboratory monitoring. Which of the following are options for ST? (Select ALL that apply.) A) Pradaxa B) Arixtra C) Eliquis D) Xarelto E) Lovenox

A) Pradaxa (Dabigatran) C) Eliquis (Apixaban) D) Xarelto (Rivaroxaban) Pradaxa, Eliquis and Xarelto are oral anticoagulant options for this patient that do not require laboratory montoring for efficacy.

While working as an inpatient clinical pharmacist, one of the job duties is to make sure that patients who need prophylaxis for venous thromboembolism (VTE) are receiving it. Which of the following conditions are risk factors for developing a VTE? (Select ALL that apply.) A) Pregnancy B) Estrogen-containing medications C) Cancer D) Major trauma or surgery E) Previous history of DVT or PE

A) Pregnancy B) Estrogen-containing medications C) Cancer D) Major trauma or surgery E) Previous history of DVT or PE Recognizing conditions can increase the risk of VTE (and could warrant VTE prophylaxis) is important.

Choose the correct antidote to use in the case of a heparin overdose: A) Protamine B) N-acetylcysteine C) Vitamin K D) Flumazenil E) Naloxone

A) Protamine Protamine is the antidote for heparin.

Which of the following medications will enhance warfarin's metabolism? (Select ALL that apply.) A) Rifampin B) Cimetidine C) St. John's Wort D) Amiodarone E) Phenytoin

A) Rifampin C) St. John's Wort E) Phenytoin Rifampin, St. John's Wort and phenytoin (among others) will enhance the metabolism of warfarin leading to a decrease in INR.

The pharmacist will counsel a patient on the correct self-administration technique for enoxaparin. Which of the following are correct counseling statements? (Select ALL that apply.) A) This medication can cause the patient to bruise and/or bleed more easily. B) Choose an area on the right or left side of the patient's abdomen, but not within two inches from the belly button. C) Do not expel the air bubble in the syringe prior to injection. D) Store this medication in the refrigerator until just prior to each use. E) It is best to rub the injection site after administration to ensure quick absorption.

A) This medication can cause the patient to bruise and/or bleed more easily. B) Choose an area on the right or left side of the patient's abdomen, but not within two inches from the belly button. C) Do not expel the air bubble in the syringe prior to injection. Do not expel the air bubble in the syringe as it can cause the patient to get a subtherapeutic dose because some of the medicine will be lost (as long as the exact dose needed is the amount in the syringe). With some medications it is recommended to rub the site after injection, but not with drugs that can cause bleeding, such as this one.

The brand name of bivalirudin is: A) Bevyxxa B) Angiomax C) Kcentra D) Iprivask E) Arixtra

B) Angiomax

The pharmacist receives a call from the medical floor. A patient on a heparin drip has been diagnosed with a GI bleed. How much heparin will 1 mg of protamine reverse? A) 1 unit B) 100 units C) 1,000 units D) 10,000 units E) 100,000 units

B) 100 units 1 mg of protamine will reverse ~100 units of heparin. The maximum dose of protamine is 50 mg. Because of the very short half-life of IV heparin, we generally only need to reverse what the patient has received in the last 2-2.5 hours. Protamine can also be used to reverse enoxaparin administered in the previous 8 hours. The dose is 1 mg protamine per 1 mg of enoxaparin.

GF is a 54 year old female with a mechanical mitral valve, hypertension, and GERD. What is the correct INR target range for GF? A) 3.0 - 4.0 B) 2.5 - 3.5 C) 2.0 - 3.0 D)1.5 - 2.5 E) Warfarin is not indicated for this patient

B) 2.5 - 3.5 Goal INR range for patients with a mechanical mitral valve is 2.5 - 3.5.

Which diagnosis increases a patient's risk of venous thromboembolism? (Select ALL that apply.) A) Wenicke-Korsakoff syndrome B) Antithrombin deficiency C) Factor V Leiden D) Antiphospholipid syndrome E) Congenital protein C deficiency

B) Antithrombin deficiency C) Factor V Leiden D) Antiphospholipid syndrome E) Congenital protein C deficiency Patients with antithrombin deficiency, Factor V Leiden, antiphospholipid syndrome and Protein C or S deficiency are prone to clotting. Many of these patients present with unprovoked clots at a relatively early age and require life-long anticoagulation. Wenicke-Korsakoff syndrome is caused by thiamine deficiency (refer to the Hepatitis & Liver Disease chapter).

BK, a 22 year old female, is admitted for an acute PE. Her past medical history is significant for antithrombin deficiency. Her allergies include cat hair and PCN (shortness of breath for both). Which of the following agents would be best to recommend for treatment of her PE? A) Heparin B) Argatroban C) Lovenox D) Arixtra E) Fragmin

B) Argatroban This patient is antithrombin deficient; therefore, heparin, LMWHs and fondaparinux will not work.

A 70 year-old patient was prescribed warfarin in the hospital for a deep vein thrombosis (DVT) in her right lower leg. She is being discharged, and the outpatient pharmacist who is going to dispense her warfarin is checking her medication profile for drug interactions. The pharmacist notes that the patient is using several medications which increase the risk of bleeding. She will counsel the patient on increased bleeding risk. Which of the following medications can increase her bleeding risk? (Select ALL that apply.) A) Co-enzyme Q10 B) Clopidogrel C) Flagyl D) Zoloft E) Carbamazepine

B) Clopidogrel C) Flagyl D) Zoloft Clopidogrel (or any antiplatelet medication), metronidazole (Flagyl) and SSRIs/SNRIs (like sertraline) can increase the risk of bleeding in patients taking warfarin. Carbamazepine is strong CYP inducer, which would increase metabolism of warfarin and decrease the INR.

A hospitalized patient developed a pulmonary embolism and was started on enoxaparin therapy. The physician began warfarin therapy on Monday and wrote an order to discontinue the enoxaparin therapy the following day. The pharmacist contacted the prescriber to recommend the following action: A) Continue the enoxaparin until one INR reading is in the therapeutic range B) Continue the enoxaparin for a minimum of 5 days and until the INR has been therapeutic for at least 24 hours. C) Continue the enoxaparin until the INR has been therapeutic for at least 72 hours. D) The enoxaparin can be stopped after a minimum of 7 days, if the INR has been therapeutic for at least 2 of those days. E) A pulmonary embolism cannot be treated with warfarin; UFH or LMWH must be used.

B) Continue the enoxaparin for a minimum of 5 days and until the INR has been therapeutic for at least 24 hours. The parenteral anticoagulant should continue for a minimum of 5 days and until the INR is therapeutic (INR at 2.0 or above in this scenario) for at least 24 hours. Both of these criteria must be met.

A nurse practitioner wishes to convert a patient from warfarin to dabigatran. She asks the pharmacist how to manage the conversion. The pharmacist should offer the following advice: A) Discontinue warfarin and start dabigatran when the INR is below 2.5. B) Discontinue warfarin and start dabigatran when the INR is at or below 2. C) Discontinue warfarin and start dabigatran when the INR is at or below 1.5. D) Stop warfarin and initiate dabigatran the following morning. E) Discontinue warfarin and start dabigatran when the INR is below 3.

B) Discontinue warfarin and start dabigatran when the INR is at or below 2. When converting patients from warfarin therapy to dabigatran, discontinue warfarin and start dabigatran when the international normalized ratio (INR) is below 2. Refer to the full prescribing information for recommendations on switching to and from warfarin and other oral anticoagulants. READ Rivaroxaban when INR is < 3 Edoxban when INR is ≤ 2.5 Apixaban when INR is < 2 Dabigatran when INR is < 2

Rivaroxaban works by the following mechanism of action: A) Vitamin K antagonist B) Factor Xa inhibitor C) Direct Factor IIa inhibitor D) PAR-1 inhibitor E) Inhibits antithombin

B) Factor Xa inhibitor Rivaroxaban is an oral Factor Xa inhibitor.

A pharmacy receives a prescription for warfarin 2.5 mg PO daily. What is the color of the warfarin tablet? A) Lavender B) Green C) Tan D) Pink E) Blue

B) Green Please Let Greg Brown Bring Peaches To Your Wedding

A 42 year-old female with a heart condition presents to the hospital with a DVT. The medical resident wishes to give her a low molecular weight heparin (LMWH), but the supervising physician insists on using unfractionated heparin. What are advantages to the use of LMWHs over unfractionated heparin? (Select ALL that apply.) A) LMWHs are more efficacious than heparin in treating DVTs. B) LMWHs are more cost effective than heparin. C) LMWHs are easier to reverse in patients that experience significant bleeding. D) LMWHs do not require monitoring in most patients. E) LMWHs have a more consistent anticoagulation response.

B) LMWHs are more cost effective than heparin. D) LMWHs do not require monitoring in most patients. E) LMWHs have a more consistent anticoagulation response. When dosed correctly, UFH and LMWH are generally thought to be equally effective for most indications. Unlike heparin, LMWHs do not require routine monitoring. Anti-factor Xa levels can be monitored in select patients, but this is not necessary in most patients. LMWHs are more cost effective than heparin, because they do not require monitoring (in most cases), offer a more predictable response and patients can be discharged earlier.

Apixaban, edoxaban, rivaroxaban and dabigatran are all approved for which of the following indications? A) Atrial fibrillation associated with mechanical heart valves and treatment of DVT/PE B) Nonvalvular atrial fibrillation and treatment of DVT/PE C) Nonvalvular atrial fibrillation, DVT treatment and DVT prophylaxis D) DVT prophylaxis after hip surgery E) DVT treatment and DVT prophylaxis

B) Nonvalvular atrial fibrillation and treatment of DVT/PE All of these medications are now approved for nonvalvular atrial fibrillation and treatment of DVT/PE. Some have other indications, but it helps to know what they have in common.

KP is a 58 year-old female who is usually well-controlled on a warfarin regimen of 5 mg daily. She has been sick for the past week but feels better today. She ate little during her illness. She presents to the anticoagulation clinic to have her INR checked. Her INR is elevated today at 5.8. There is no noticeable bleeding and she is at low risk of bleeding. Choose the preferred course of action: A) Hold warfarin x 1 and administer phytonadione 1 to 2.5 mg orally. B) Omit the next few doses, monitor frequently, and resume therapy at a lower dose when the INR is in the therapeutic range. C) Hold warfarin and administer phytonadione 2 mg by SC injection. D) Hold warfarin and administer vitamin K 5 mg orally. E) Hold warfarin and administer phytonadione 2 mg by IM injection.

B) Omit the next few doses, monitor frequently, and resume therapy at a lower dose when the INR is in the therapeutic range. According to the CHEST 2012 guidelines, patients with a supratherapeutic INR of 4.5 - 10 and without bleeding should not routinely receive vitamin K. Therefore, the patient should have 1 - 2 doses of warfarin held and the INR monitored. Restart warfarin at a lower dose when the INR is in the therapeutic range.

Which of the following is a possible side effect from the long-term use of heparin therapy? A) Gingival hyperplasia B) Osteoporosis C) GERD D) Hair growth E) Hypokalemia

B) Osteoporosis It is rare to see patients on heparin for extended periods of time, but osteoporosis can occur with long-term use. Women who are pregnant and use heparin throughout their pregnancy are at risk for decreased bone density.

Which of the following are signs/symptoms that could indicate a patient is bleeding? (Select ALL that apply.) A) Xerostomia B) Red or black stools C) Epistaxis D) Metallic taste in mouth when she brushes her teeth E) Ecchymosis

B) Red or black stools C) Epistaxis D) Metallic taste in mouth when she brushes her teeth E) Ecchymosis Epistaxis is the technical term for a nose bleed. Ecchymosis is bruising. Blood contains iron and tastes metallic. If a patient's gums are bleeding when they are brushing their teeth, they may notice this metallic taste. Other signs of bleeding could include pain, swelling, or discomfort, bleeding from cuts that takes a long time to stop (usually more than 15 minutes), menstrual bleeding or vaginal bleeding that is much heavier than normal, pink or brown urine (or dark, tarry stools) or vomiting blood or material that looks like coffee grounds (hematemesis). Dark tarry stools are more likely with NSAID-induced bleeding, but would be aggravated by an elevated INR.

SS went to an urgent care center for treatment of cellulitis. She was prescribed Septra DS PO BID x 10 days. How will the Septra DS affect her warfarin therapy? A) Septra DS is contraindicated with warfarin. Instruct patient not to take the Septra DS. B) Septra DS interacts with warfarin by increasing the INR. Recommend decreasing the warfarin dose and monitor closely. C) Septra DS interacts with warfarin by decreasing the INR. Recommend increasing the warfarin dose by 10-20% and monitor closely. D) Septra DS interacts with warfarin by decreasing the INR. Recommend increasing the warfarin dose by 30-50% and monitor closely. E) Septra DS does not interact with warfarin.

B) Septra DS interacts with warfarin by increasing the INR. Recommend decreasing the warfarin dose and monitor closely. Sulfamethoxazole/trimethoprim (Bactrim, Septra) impairs the hepatic metabolism of warfarin. It is a strong 2C9 inhibitor, and may also involve displacement of warfarin from protein binding sites or alterations in intestinal flora. This interaction can cause the INR to increase.

A pharmacist working in an inpatient medical ward of the local hospital is responsible for monitoring anticoagulation therapy. She routinely obtains laboratory parameters and adjusts the doses of low molecular weight heparins (LMWHs), as needed. In which of the following clinical situations is it appropriate to monitor the level of anticoagulation with LMWH therapy? (Select ALL that apply.) A) Patients with a myocardial infarction B) Significant renal impairment C) Pregnant patient with PE D) Pregnant patient with mechanical heart valves E) Extremes of body weight

B) Significant renal impairment C) Pregnant patient with PE D) Pregnant patient with mechanical heart valves E) Extremes of body weight These clinical scenarios are appropriate for monitoring LMWH therapy.

Low molecular weight heparins have a boxed warning concerning this risk: A) Stevens Johnson syndrome B) Spinal or epidural hematoma formation C) Pancreatitis D) Severe bleeding E) Acute renal failure

B) Spinal or epidural hematoma formation LMWHs, the Xa inhibitors and dabigatran all carry this boxed warning. The risk is highest if the patient receives neuraxial anesthesia or has a spinal puncture concurrently. These hematomas may result in long-term or permanent paralysis.

LA has been started on Lovenox 90 mg SC Q12H for ACS. She is stable from a cardiac standpoint, with a negative ECG and negative cardiac enzymes. She has received 3 doses of Lovenox, the last dose was given 4 hours ago. She now has significant bleeding from her IV lines. Which of the following are appropriate measures to initiate? (Select ALL that apply.) A) Administer phytonadione now B) Stop Lovenox therapy C) Administer protamine now D) Start argatroban therapy E) Call code blue

B) Stop Lovenox therapy C) Administer protamine now The patient is experiencing significant bleeding. Discontinue Lovenox (at least temporarily) and give protamine which will neutralize ~60% of the anti-Factor Xa activity of Lovenox.

A patient has been using warfarin therapy for several years. He does not like the time it takes to report for lab monitoring. He asks if he can have his INR checked every six months. The ACCP guidelines recommend the following monitoring schedule for patients who are well-controlled on a stable dose of warfarin: A) The INR should be checked at least weekly in all patients. B) The INR can be checked at up to 12 week intervals in stable patients. C) The INR should be checked monthly in all patients. D) The INR should be checked at least every 4 months in stable patients. E) The INR should be checked at least every 6 months in stable patients.

B) The INR can be checked at up to 12 week intervals in stable patients. Once controlled on a stable dose of warfarin for a reasonable time period, the INR can be monitored at up to a 12 week interval.

What would be expected to occur if a patient on warfarin with a stable INR is started on fluconazole? A) The INR would increase and the patient may clot. B) The INR would increase and the patient may experience bleeding. C) The INR would decrease and the patient may clot. D) The INR would decrease and the patient may experience bleeding. E) The INR would not change.

B) The INR would increase and the patient may experience bleeding. Fluconazole inhibits the metabolism of warfarin; therefore, increasing the INR and potentially causing the patient to bleed.

CD is a 42 year-old male with type 2 diabetes, hypertension and high cholesterol. He did not practice proper foot care and developed an infected big toe. Unfortunately, the infection spread into the bone and he was admitted to the hospital and had his toe amputated. While resting in bed after the surgery, CD developed a deep vein thrombosis. His creatinine clearance is estimated at 55 mL/min and his weight is 80 kg. The physician orders enoxaparin 80 mg SC BID for DVT treatment. Choose the correct statement: A) The dose is correct as ordered. B) The dose should be 80 mg SC daily. C) The dose should be 160 mg SC BID. D) The dose should be 30 mg IV bolus followed by 80 mg SC daily. E) The patient should receive unfractionated heparin.

B) The dose should be 80 mg SC daily. The ordered dose is correct based on the patient's weight and renal function. VTE Treatment: 1 mg/kg SC Q12H or 1.5 mg/kg SC daily

Which of the following should be discussed with a patient receiving a new prescription for Pradaxa? A) The generic name of this medication is rivaroxaban. B) This medication must be kept in the original container. Do not put into a pill box. C) This medication requires periodic laboratory monitoring. D) This medication is used to prevent blood clots around your artificial heart valve. E) Take this medication with food.

B) This medication must be kept in the original container. Do not put into a pill box. Dabigatran (Pradaxa) must be kept in the original container and discarded 4 months after opening the original container. Capsules must be swallowed whole.

Which of the following statements regarding warfarin is/are correct? (Select ALL that apply.) A) The antidote is protamine. B) Warfarin blocks the activation of clotting factors II, VII, IX, and X. C) Patients on warfarin should not eat any vitamin K containing foods. D) Warfarin is a vitamin K antagonist. E) Warfarin generally takes 2 days to become therapeutic.

B) Warfarin blocks the activation of clotting factors II, VII, IX, and X. D) Warfarin is a vitamin K antagonist. Warfarin is a vitamin K antagonist. Vitamin K is required for the carboxylation of clotting factors II, VII, IX, and X. Without adequate vitamin K, the liver produces the factors, but they have reduced coagulant activity.

Which drug/s act via direct inhibition of Factor Xa? (Select ALL that apply.) A) Pradaxa B) Xarelto C) Savaysa D) Praxbind E) Eliquis

B) Xarelto C) Savaysa E) Eliquis

SK is a 55 year-old female with a mechanical aortic heart valve, atrial fibrillation, and osteoporosis. What is the recommended therapeutic INR range of warfarin for this patient? A) 3.5-4.5 B) 2.5-3.5 C) 2-3 D) 1.5-2.5 E) 3-4

C) 2-3 According to the 2012 CHEST guidelines, patients with a mechanical aortic valve should have an INR between 2-3. Patients with 2 mechanical valves or a single mechanical mitral valve require a higher INR goal (2.5-3.5) because these scenarios are associated with greater risk of clotting.

A 55 year old man (weighing 222 lbs) comes in to the emergency department with crushing chest pain. His creatinine clearance is estimated at 45 mL/min. The physician makes the diagnosis of ST-segment elevation myocardial infarction (STEMI) and wants to initiate enoxaparin right away. What is the correct dose of enoxaparin in this patient? A) 100 mg SC every 12 hours started immediately B) 100 mg SC every day sarted immediately C) 30 mg IV bolus plus 100 mg SC every 12 hours started immediately after the bolus D) 220 mg SC every day started immediately E) This patient is not a candidate for enoxaparin therapy

C) 30 mg IV bolus plus 100 mg SC every 12 hours started immediately after the bolus In patients with STEMI who are less than 75 years old, give enoxaparin 30 mg IV bolus plus a 1 mg/kg SC dose followed by 1 mg/kg SC dose Q12H (max 100 mg for the first two doses) in patients with CrCl > 30 mL/min. This is the only clinical scenario where it is recommended to administer enoxaparin as an IV bolus.

A patient is being started on warfarin therapy. Which of the following scenarios would pose an increased risk of bleeding from warfarin? A) Also taking a 2B6 inhibitor B) Also taking a 2D6 inhibitor C) Also taking a 2C9 inhibitor D) Also taking a 2D6 inducer E) Also taking a 2C9 inducer

C) Also taking a 2C9 inhibitor A major 2C9 inhibitor will reduce the metabolism of warfarin, increasing the risk of bleeding. A 2C9 inducer would increase warfarin metabolism, increasing the risk of clotting. 2C9 inhibitors and inducers can be reviewed in the Anticoagulation chapter as well as the Drug Interactions chapter of the RxPrep Course Book.

Which of the following is the most likely adverse effect from the use of heparin? A) Leukopenia B) Hypercalcemia C) Bleeding D) Decreased cognitive function E) Lupus like syndrome

C) Bleeding The major side effect of any anticoagulant is bleeding.

LE is taking warfarin 2.5 mg daily and Lovenox 80 mg SC Q12H for a DVT diagnosed 5 days ago. On Day 4 of her treatment, LE fell on her way to the grocery store. She is now in the ED with a left leg hematoma and a few lacerations but otherwise hemodynamically stable. The ED resident calls the pharmacist-managed anticoagulation services for advice on LE's anticoagulation management for her DVT treatment. Her INR today is 1.8. What should the anticoagulation pharmacist recommend regarding LE's therapy? A) Discontinue Lovenox. Hold warfarin for 2 days until hematoma improves. B) Discontinue Lovenox. Continue warfarin. C) Continue Lovenox and warfarin. D) Admit patient to the hospital for UFH therapy. E) Discontinue Lovenox and warfarin.

C) Continue Lovenox and warfarin. As long as the patient is hemodynamically stable with no sign or symptoms of uncontrollable bleeding, the patient should continue to receive LMWH and warfarin therapy as directed for recent DVT treatment. It would be appropriate to educate on local measures to stop bleeding from the leg hematoma or lacerations. Since the INR is not yet therapeutic, the Lovenox must be continued.

The pharmacist is counseling AB, a patient beginning warfarin therapy. The pharmacist informs AB that if his toes become painful and/or purple in color, he must contact his healthcare provider right away. If purple toe develops, the healthcare provider should: A) Decrease the warfarin dose B) Increase the warfarin dose C) Discontinue warfarin D) Suggest warm compresses be applied to the area E) Change warfarin to unfractionated heparin

C) Discontinue warfarin Although the frequency of purple toe syndrome is low, the number of patients receiving warfarin is high. Pharmacists should maintain vigilance when counseling patients who are taking warfarin, especially those with atherosclerotic disease who have highest risk for purple toe syndrome. If present, the patient must report to the physician and warfarin will be discontinued.

A patient has used warfarin daily for many years. She usually takes the warfarin at 6:00 PM with dinner. She calls the pharmacy because she got stuck in a snowstorm yesterday and could not return home to take her warfarin dose last night. It is now 4:00 PM the following day. She asks the pharmacist if she should take yesterday's warfarin dose now. The pharmacist should offer the following advice: A) You can take both doses with dinner tonight. B) Take yesterday's dose now and take today's dose at the normal time. C) Do not take yesterday's dose; missed doses should not be taken on the following day. D) Contact the physician at once for an emergency check of the INR. E) Take 1/2 of the missed dose now and take today's dose at the normal time.

C) Do not take yesterday's dose; missed doses should not be taken on the following day. Instruct patients that if they miss a dose, take the dose as soon as possible on the same day. Do not take a double dose the next day to make up for a missed dose.

CJ is a 44 year-old female with hyperlipidemia and hypertension. She is referred to a pharmacist-managed anticoagulation clinic with a newly diagnosed, first unprovoked episode of lower left leg DVT. She is a teacher and drinks 1 glass of wine per day. Lab/vitals: Weight: 325 lbs, Height: 6'1", Scr 0.6 mg/dL (1 month ago), Baseline INR 1.1 Which of the following is the correct enoxaparin dose for CJ? A) Enoxaparin 220 mg SC BID B) Enoxaparin 150 mg SC daily C) Enoxaparin 150 mg SC Q12H D) Enoxaparin 100 mg SC daily E) Enoxaparin 100 mg SC Q12H

C) Enoxaparin 150 mg SC Q12H Enoxaparin is dosed based on actual body weight.

JD is a 76 year-old male with a mechanical mitral heart valve, hypertension, type 2 diabetes, and hyperlipidemia. JD will be undergoing an elective major surgical procedure and he is considered high-risk for thromboembolism. Medications: Zestril Mevacor Glucotrol XL Jantoven Coumadin Labs: SCr 0.85 mg/dL Weight 200 lbs Height 5'7" INR 2.9 What is the best peri-operative anticoagulation plan for this patient regarding his warfarin therapy? A) Hold warfarin approximately 5 days prior to procedure. No bridging with LMWH needed. B) Hold warfarin approximately 5 days prior to procedure. Bridge with Lovenox 40 mg IV daily. C) Hold warfarin approximately 5 days prior to procedure. Bridge with Lovenox 90 mg SC twice daily. D) Hold warfarin approximately 1 day prior to procedure. Bridge with Lovenox 90 mg SC BID. E) Hold warfarin for 2 days prior to procedure. Bridge with Lovenox 110 mg SC daily.

C) Hold warfarin approximately 5 days prior to procedure. Bridge with Lovenox 90 mg SC twice daily. Patients at high risk for thromboembolism should be bridged with a therapeutic dose of enoxaparin. The enoxaparin would be stopped 24 hours before the surgery.

CJ is a 44 year-old female with hyperlipidemia and hypertension. She is referred to a pharmacist-managed anticoagulation clinic with a newly diagnosed, first unprovoked episode of lower left leg DVT. She is a teacher and drinks 1 glass of wine per day. Lab/vitals: Weight: 325 lbs, Height: 6'1", Scr 0.6 mg/dL (1 month ago), Baseline INR 1.1. What INR goal and duration of warfarin therapy should be recommended to treat CJ's DVT? A) INR goal of 1.5-2.5 for 6 months B) INR goal of 2-3 for 1 year C) INR goal of 2-3 for at least 3 months D) INR goal of 2-3 indefinitely E) INR goal of 2.5-3.5 for 3-6 months

C) INR goal of 2-3 for at least 3 months Guidelines recommend that the first, unprovoked VTE be treated for at least 3 months with an INR goal of 2-3.

Which of the following statements regarding Kcentra are correct? (Select ALL that apply.) A) Kcentra can cause severe bleeding. B) Kcentra is indicated for HIT. C) Kcentra should be administered with vitamin K . D) Kcentra is indicated to reverse warfarin in cases of acute bleeding or need for urgent surgery. E) Kcentra contains factors II, VII, IX, X, protein C and protein S.

C) Kcentra should be administered with vitamin K . D) Kcentra is indicated to reverse warfarin in cases of acute bleeding or need for urgent surgery. E) Kcentra contains factors II, VII, IX, X, protein C and protein S. Kcentra should not be used in HIT as it contains heparin in the formulation. Kcentra can cause clotting.

The physician plans to start ST on enoxaparin, but would like to order a laboratory test to monitor efficacy of enoxaparin therapy. Which of the following could be recommended? A) aPTT Q6H B) INR daily C) Peak anti-Xa, 4 hours after the dose D) Trough anti-Xa, before the next dose E) There is no laboratory test to monitor efficacy of this therapy

C) Peak anti-Xa, 4 hours after the dose Routine monitoring of enoxaparin with anti-Xa levels is not necessary, but can be done in certain patients. Anti-Xa levels should be drawn 4 hrs after the SQ dose (peak).

Which lab value must be carefully monitored when a patient is receiving heparin? A) White blood cells B) Eosinophils C) Platelets D) Amylase E) Sodium

C) Platelets A serious adverse effect associated with heparin therapy is heparin-induced thrombocytopenia (HIT). This is a significant drop in platelets caused by an immune response against platelets. Platelets must be monitored during therapy.

A 42 year-old female patient with no known medical conditions has just returned from a trip to Taiwan. The patient is found to have a DVT and warfarin therapy is to be initiated as an outpatient. What is the most appropriate dosing regimen to begin warfarin in this patient? A) The patient is required to have genetic testing done first to help determine the best dose. B) Start the warfarin at 2.5 mg for the first 1-2 days, then adjust per the INR value. C) Start the warfarin at 10 mg for the first 1-2 days, then adjust per the INR value. D) Start the warfarin at 20 mg for the first day, 10 mg on day 2, then adjust per the INR value. E) Start warfarin at 2.5 mg for the first day, 5 mg on day 2, then adjust per the INR value.

C) Start the warfarin at 10 mg for the first 1-2 days, then adjust per the INR value. Genetic testing for warfarin is not routinely recommended, according to the CHEST 2012 updated recommendations. A starting dose of 10 mg for the first 2 doses (then adjust per INR value) is appropriate for patients who are healthy enough to begin therapy as outpatients.

Select the correct dosing recommendation for dabigatran for a patient with a DVT and a creatinine clearance of 54 mL/min: A) Take a 150 mg capsule twice daily, with food. B) Take a 150 mg capsule twice daily, without food. C) Take a 150 mg capsule twice daily, with or without food. D) Take a 75 mg capsule twice daily, with or without food. E) Take 150 mg capsule once daily, with food.

C) Take a 150 mg capsule twice daily, with or without food. The standard dose of dabigatran is 150 mg PO BID for non-valvular AF and DVT/PE treatment in patients with good renal function. For patients with atrial fibrillation and CrCl 15-30 mL/min, the recommended dose is 75 mg twice daily. Dabigatran is taken without regard to meals.

Which of the following organizations sets the guidelines for the management of antithrombotics? A) The American Society of Hypertension Physicians, published in JNC 8 B) The American Association of Clinical Endocrinologists (AACE), published in the journal AACE C) The American College of Chest Physicians (ACCP): Evidence-Based Clinical Practice Guidelines, published in the journal CHEST D) The American Society of Cardiology Physicians: Anticoagulation Practice Guidelines, published in the journal Coagulation E) The American College of Surgeons (ACS): Guidelines for Anticoagulation Reversal, published in the journal Surgery

C) The American College of Chest Physicians (ACCP): Evidence-Based Clinical Practice Guidelines, published in the journal CHEST The CHEST guidelines are used for antithrombotics. There are valuable resources on the safe use of antithrombotics on the Institute for Safe Medication Practices at www.ismp.org

What effect would be predicted if a patient on warfarin with a stable INR is started on amiodarone? A) The INR would decrease and the patient may experience bleeding. B) The INR would increase and the patient may clot. C) The INR would increase and the patient may experience bleeding. D) The INR would decrease and the patient may clot. E) The INR would not change.

C) The INR would increase and the patient may experience bleeding. Amiodarone inhibits the metabolism of warfarin; therefore, increasing the INR and potentially causing the patient to bleed. When a patient stabilized on warfarin will require amiodarone, clinicians generally reduce the warfarin dose by 30-50% to account for this.

What is the purpose of using a heparin "lock-flush," such as HepFlush? A) To provide prophylaxis in patients at risk of stroke (e.g., patients with atrial fibrillation) B) To provide systemic anticoagulation treatment C) To keep IV lines open D) To prevent HIT E) To dilute other medications going through the same IV line

C) To keep IV lines open Heparin "lock-flushes" (HepFlush) are used to keep IV lines open (patent). They are not used for anticoagulation. There have been fatal errors made by choosing the incorrect heparin strength. Using a higher dose to flush a line could cause significant bleeding, including fatal hemorrhage. Many of the dosing errors have occurred in neonates.

What duration of anticoagulation is recommended after cardioversion for atrial fibrillation? A) 1 week B) 2 weeks C) 3 weeks D) 4 weeks E) Patients do not require anticoagulation after cardioversion

D) 4 weeks Regardless of the duration of atrial fibrillation or the method of cardioversion, a minimum of 4 weeks of anticoagulation is recommended afterward.

A 77 year old patient with atrial fibrillation, hypertension, Crohn's disease and heart failure is seen in clinic today. Her medications include Norvasc, Entocort EC, Diovan, Coreg, and Lasix. In order to optimize stroke prevention, the following recommendation should be made to the physician overseeing her care: A) Start this patient on aspirin therapy, based on CHADS2 score. B) Discontinue Norvasc since this can increase the risk of stroke. C) This patient is not a candidate for warfarin therapy due to significant drug interactions. D) Add warfarin therapy to this patient's regimen, based on CHADS2 score. E) Discontinue Diovan since this can increase the risk of stroke.

D) Add warfarin therapy to this patient's regimen, based on CHADS2 score. The patient's CHADS2 score is 3. Oral anticoagulation is recommended for patients wth scores ≥ 2.

Vitamin K given IV has a risk of the following adverse reaction: A) Acute dystrophy B) Seizures C) Peripheral neuropathy D) Anaphylaxis E) Neuroleptic malignant syndrome

D) Anaphylaxis The administration of vitamin K intravenously is associated with the risk of anaphylaxis (dyspnea, cardiac arrest, hypotension, shock).

The CHADS2 scoring scale assesses stroke risk in patients with: A) Venous thromboembolism B) Mitral valve replacement C) Previous ischemic strokes D) Atrial fibrillation E) Hypertension

D) Atrial fibrillation CHADS2 scoring system is for patients with atrial fibrillation to help guide therapy for the prevention of ischemic strokes.

A patient has developed a DVT and will be placed on enoxaparin. Choose the correct statement concerning enoxaparin: A) Enoxaparin is safe to use if a person has a history of heparin-induced thrombocytopenia. B) Enoxaparin is safe to use in a patient receiving concurrent neuraxial anesthesia. C) Enoxaparin is administered by intramuscular injection. D) Enoxaparin can be administred by the patient at home in many circumstances. E) Enoxaparin cannot be used safely in a patient with a sulfa allergy.

D) Enoxaparin can be administred by the patient at home in many circumstances. Enoxaparin is administered by subcutaneous (SC) injection and is contraindicated if the patient had a history of HIT. Many patients with acute DVT can be counseled on how to administer the injections and discharged from the hospital earlier.

Which of the following groups of laboratory parameters need to be monitored during heparin therapy? A) Hematocrit, hemogloblin, platelets, and PT B) Hematocrit, hemoglobin, platelets, AST, and ALT C) SCr, platelets, aPTT, and PT D) Hematocrit, hemoglobin, platelets, and aPTT E) Platelets, aPTT, PT, and SCr

D) Hematocrit, hemoglobin, platelets, and aPTT Hematocrit, hemoglobin, platelets, and aPTT are important laboratory parameters to monitor while a patient is receiving heparin therapy.

A female patient who is pregnant has been admitted to the hospital with a DVT. The physician will begin heparin therapy. What is the mechanism of action of heparin? A) Heparin potentiates factor V B) Heparin potentiates factor IXa C) Heparin potentiates factor Xa D) Heparin potentiates antithrombin E) Heparin inhibits clotting factors II, VII, IX, & X

D) Heparin potentiates antithrombin Heparin binds to antithrombin and then inactivates Factor IIa (thrombin) and Factor Xa.

HY is a 58 year-old male with atrial fibrillation. He has been using warfarin for over two years and is normally well-controlled. His cardiologist recently prescribed amiodarone and citalopram therapy with no other medication adjustments. He is admitted to the emergency room with weakness and bleeding gums. His INR is 9.5 and hemoglobin is 8.4 g/dL. He reports bright red blood in his stool which started this morning. Choose the correct course of action: A) Hold warfarin x 1 dose and administer phytonadione 1 to 2.5 mg orally. B) Omit the next 1-2 doses, monitor frequently, and resume therapy when the INR is in the therapeutic range. C) Hold warfarin and administer phytonadione 2 mg by SC injection. D) Hold warfarin therapy and give vitamin K 10 mg by slow IV injection along with four-factor prothrombin complex concentrate. E) Hold warfarin therapy and give vitamin K 10 mg by IM injection and fresh frozen plasma.

D) Hold warfarin therapy and give vitamin K 10 mg by slow IV injection along with four-factor prothrombin complex concentrate. With major bleeding, vitamin K 10 mg should be given by slow IV injection along with four-factor prothrombin complex concentrate, which is preferred over fresh frozen plasma.

Select the correct mechanism of action for Lovenox: A) Oral direct thrombin inhibitor B) Injectable direct thrombin inhibitor C) Vitamin K antagonist D) Inhibits Factor Xa and Factor IIa via antithrombin E) Selectively inhibits Factor Xa

D) Inhibits Factor Xa and Factor IIa via antithrombin Enoxaparin (Lovenox) is a low molecular weight heparin (LMWH). LMWHs work by binding to antithrombin and inhibiting clotting factors Xa (preferentially) and IIa.

SD had an intracerebral hemorrhage ten days ago secondary to treatment for a cerebrovascular accident. She is considered to be high risk for venous thromboembolism (VTE). What is the best recommendation for preventing VTE in this patient? A) Unfractionated heparin SC BID B) Aspirin only C) Low molecular weight heparin SC BID D) Intermittent pneumatic compression devices E) Apixaban PO BID

D) Intermittent pneumatic compression devices Intermittent pneumatic compression devices are recommended for patients with high-bleeding risk. They help keep the blood moving and reduce the risk of clot formation.

The pharmacist receives a prescription for dalteparin. What type of medication is dalteparin? A) Antidote for UFH/LMWH B) Direct thrombin inhibitor C) Factor Xa inhibitor D) Low molecular weight heparin E) Vitamin K antagonist

D) Low molecular weight heparin Dalteparin (Fragmin) is a LMWH.

A patient is being started on Pradaxa. Choose the correct statement regarding Pradaxa: A) Once a bottle of Pradaxa is opened, the capsules must be used within 15 days. B) Once a bottle of Pradaxa is opened, the capsules must be used within 60 days. C) Once a bottle of Pradaxa is opened, the capsules must be used within 90 days. D) Once a bottle of Pradaxa is opened, the capsules must be used within 120 days. E) If Pradaxa capsules are transferred to an amber container, they are good up to 2 months.

D) Once a bottle of Pradaxa is opened, the capsules must be used within 120 days. The 60-count bottles of Pradaxa expire 4 months after opening the bottle.

MH is a 66 year-old male with hypertension, renal disease and degenerative joint disease. In his younger years, MH was a football player and has lived with the pain of a hip injury for many years. He enters the hospital for elective hip replacement surgery. His creatinine clearance is ~20 mL/min. The physician orders enoxaparin 30 mg SC BID for DVT prophylaxis. Choose the correct statement: A) The dose is correct as ordered. B) The dose should be 60 mg SC daily. C) The dose should be 45 mg SC daily. D) The dose should be 30 mg SC daily. E) The patient does not require DVT prophyalxis.

D) The dose should be 30 mg SC daily. The physician ordered the correct prophylactic dose of enoxaparin for a patient without significant renal disease (30 mg SC BID, or 40 mg SC daily). If the creatinine clearance is less than 30 mL/min, the dose is reduced to 30 mg SC once daily.

Select the incorrect statement about Pradaxa. A) When used for treating a DVT/PE, it can be started after 5-10 days of parenteral anticoagulation. B) The capsules should be swallowed whole; do not open, crush chew. C) It is associated with more GI bleeding than warfarin. D) The effect can be reversed with Mephyton. E) It does not require blood testing to monitor for effectiveness.

D) The effect can be reversed with Mephyton. Dabigatran (Pradaxa) is formulated as a capsule. The capsules must be swallowed whole; do not open and administer via an NG tube. Phytonadione (Mephyton) is the antidote for warfarin. The antidote for dabigatran is idarucizumab (Praxbind).

What is the name of the test used to monitor warfarin efficacy and toxicity? A) Potentiation factor B) Factors II, VII, IX and X C) Anti-Xa level D) The international normalized ratio E) Activated partial thromboplastin time

D) The international normalized ratio The international normalized ratio (INR) is the test used to measure the effectiveness of warfarin.

Patients may use the following non-pharmacological method to reduce the risk of venous thromboembolism: A) Increase their intake of green leafy vegetables. B) Perform several reps of 10 deep squats daily, if the physician approves this type of exercise. C) Consume lots of water. D) Use intermittent pneumatic compression devices. E) Consume more olive oil and green tea.

D) Use intermittent pneumatic compression devices. Graduated compression stockings (GCS) and intermittent pneumatic compression (IPC) devices are two non-pharmacologic measures used to prevent venous thromboembolism.

A 25 year-old female is receiving warfarin for a DVT. Her counseling should include the following: A) Warfarin is safe in pregnancy; there are no restrictions to using it. B) Warfarin may be safe in certain trimesters of pregnancy. C) Warfarin may be unsafe in pregnancy based on animal studies. D) Warfarin is unsafe in pregnancy but can be used if the patient refuses other treatments. E) Warfarin is unsafe in pregnancy and should be avoided.

E) Warfarin is unsafe in pregnancy and should be avoided. Warfarin is contraindicated in pregnancy (Pregnancy Category X) unless the patient has a mechanical heart valve (Pregnancy Category D).

HF is receiving a heparin drip. What is the name of the test used to monitor heparin for efficacy? A) Potentiation factor B) Factors IIa, VIIa, IXa and Xa test C) Anti-XIa D) International normalized ratio E) Activated partial thromboplastin time

E) Activated partial thromboplastin time The activated partial thromboplastin time (aPTT) is used to monitor the effect of heparin. The aPTT is the time, in seconds, for plasma to clot. A normal aPTT is generally between 22-38 seconds. The therapeutic aPTT range is determined individually for each hospital or laboratory depending on the reagent used for the test. Heparin activity can also be assessed with anti-Xa levels, though this is a newer practice in U.S. hospitals.

A long distance traveler comes into the pharmacy requesting information on strategies to reduce his clotting risk. He suffered from a DVT 2 years ago and is worried it will happen again. An appropriate recommendation to reduce VTE risk is: A) Start taking aspirin 81 mg daily. B) Start taking aspirin 325 mg daily. C) Obtain a prescription for rivaroxaban. D) Obtain a prescription for warfarin. E) Do calf muscle exercises on the plane.

E) Do calf muscle exercises on the plane. Guidelines do not recommend aspirin or anticoagulation for travelers, even those at risk for VTE. This is further discussed in the RxPrep Course Book.

Which of SN's lab results suggests that she could experience altered response to warfarin? A) Hyperglycemia B) Increased BUN/SCr C) Hypokalemia D) Hyponatremia E) Hypoalbuminemia

E) Hypoalbuminemia Warfarin is highly protein bound. Patients with low albumin (possibly malnourished) will have more free drug in the system and be at increased risk of bleeding (especially if other risks are present). These patients generally require a lower dose. Monitor carefully.

By what routes of administration can heparin be given? A) Intravenous and buccal administration B) Intravenous and intramuscular administration C) Intravenous, intramuscular and oral administration D) Intravenous, intramuscular and subcutaneous administration E) Intravenous and subcutaneous administration

E) Intravenous and subcutaneous administration Heparin is administered by IV or SC injection. Heparin is not administered by IM injection due to pain and the risk of hematoma formation. Heparin is not bioavailable via the oral route.

Which of the following anticoagulants can be used to treat patients with prosthetic heart valves? A) Eliquis B) Xarelto C) Apixaban D) Pradaxa E) Jantoven

E) Jantoven The newer agents should not be used for patients with prosthetic heart valves. Only warfarin (Jantoven) is used for treating these patients.

STs healthcare provider would like to start warfarin per the Anticoagulation Management protocol at the hospital. Which of the following is appropriate to start along with warfarin on day #1 of therapy for the DVT? A) Heparin 5000 units SQ Q8H B) Xarelto 20 mg daily C) Eliquis 2.5 mg BID D) Desirudin 15 mg SC Q12H E) Lovenox 90 mg SC Q12H

E) Lovenox 90 mg SC Q12H Warfarin should be started on the same day as a parenteral anticoagulant (LMWH or UFH) for DVT/PE. The parenteral anticoagulant must be used in a treatment dose, not a prophylactic dose.

FF is starting Xarelto for an acute DVT of the left lower extremity. Which of the following medications should be started on day #1 along with Xarelto? A) Warfarin B) Aspirin C) Enoxaparin D) Fondaparinux E) No other medication is required.

E) No other medication is required. Xarelto, Pradaxa, Eliquis and Savaysa provide rapid systemic anticoagulation once they are taken. They are not used simultaneously with parenteral anticoagulation. When treating an acute DVT with dabigatran or edoxaban the labeling for each drug states that these are approved for use after 5-10 days of parenteral anticoagulant. Rivaroxaban and apixaban do not have this stipulation in their labeling.

While reviewing a patient's chart, the pharmacist sees the following note: "Send tests for VKORC1 and CYP2C9 alleles" Which medication do these tests relate to? A) Apixaban B) Betrixaban C) Enoxaparin D) Fondaparinux E) Warfarin

E) Warfarin Pharmacogenomic testing for warfarin is available, but not routinely performed. Presence of the CYP 2C92 or 3 alleles and/or polymorphism of the VKORC1 gene can increase bleeding risk.


Related study sets

Midterms - Organizational Behavior

View Set

REE 4433 Exam 2 (Bailey) with case studies version 2

View Set

History Chapter 22 & 23 Test Study Guide

View Set

NUR 312 - Week 3 - ch. 41, 41, gid 2, gid 26, gid 27

View Set

Maternal-Newborn ch 20 Conditions Occurring During Pregnancy

View Set

Week 5: Drugs for Gastrointestinal Problems

View Set

Chapter 15: Oncology: Nursing Management in Cancer Care NCLEX

View Set

Term 1: Chapter 9: The Market Revolution ★ 1800-1840

View Set